Very intresting CR - Must be true

This topic has expert replies
Legendary Member
Posts: 857
Joined: Wed Aug 25, 2010 1:36 am
Thanked: 56 times
Followed by:15 members

Very intresting CR - Must be true

by AIM GMAT » Mon Apr 18, 2011 8:37 am
No mathematician today would flatly refuse to accept the results of an enormous computation as an adequate demonstration of the truth of a theorem. In 1976, however, this was not the case. Some mathematicians at that time refused to accept the results of a complex computer demonstration of a very simple mapping theorem. Although some mathematicians still hold a strong belief that a simple theorem ought to have a short, simple proof, in fact, some simple theorems have required enormous proofs.

If all of the statements in the passage are true, which one of the following must also be true?

(A) Today, some mathematicians who believe that a simple theorem ought to have a simple proof would consider accepting the results of an enormous computation as a demonstration of the truth of a theorem.
(B) Some individuals who believe that a simple theorem ought to have a simple proof are not mathematicians.
(C) Today, some individuals who refuse to accept the results of an enormous computation as a demonstration of the truth of a theorem believe that a simple theorem ought to have a simple proof.
(D) Some individuals who do not believe that a simple theorem ought to have a simple proof would not be willing to accept the results of an enormous computation as proof of a complex theorem.
(E) Some nonmathematicians do not believe that a simple theorem ought to have a simple proof.
Thanks & Regards,
AIM GMAT

User avatar
Legendary Member
Posts: 582
Joined: Tue Mar 08, 2011 12:48 am
Thanked: 61 times
Followed by:6 members
GMAT Score:740

by force5 » Mon Apr 18, 2011 1:17 pm
@ Aim where did u get this CR from?? i needed a cup of coffee after this.....

Tried my best and was left with A

important sentence is
No mathematician today would flatly refuse to accept the results of an enormous computation as an adequate demonstration of the truth of a theorem.
Hence A

User avatar
GMAT Instructor
Posts: 1031
Joined: Thu Jul 03, 2008 1:23 pm
Location: Malibu, CA
Thanked: 716 times
Followed by:255 members
GMAT Score:750

by Brian@VeritasPrep » Mon Apr 18, 2011 4:52 pm
Wow - yeah, great question here! This is as involved (if not more so) than anything I think you'd see on the GMAT and I think it may rely more on formal logic than a typical GMAT question, too, so I'd be surprised if this one didn't come from the LSAT.

The way I looked at this one, the premises ONLY tell us about mathematicians; we don't explicitly know anything about anyone else. So unless "some individuals" in B, C, and D include mathematicians as part of those groups, they won't work...they're already a little suspect to me. And E is out - we don't know anything about them.

B also focuses (in the second half) on non-mathematicians, so we don't know about them. Same with D.

C, at least to me, is a little tricky, but again it goes back to the fact that we don't know anything about non-mathematicians. And since "some who refuse to accept the results of an enormous computation" rules out (based on sentence 1 of the stimulus) all mathematicians, we don't know anything about anyone else and we can't prove C.

A is left, and it's a pretty nice paraphrase of the first sentence of the paragraph. We know that "no mathematician would not accept..." the complex results, so it's safe to say that some mathematicians who have a certain belief would consider accepting those results.
Brian Galvin
GMAT Instructor
Chief Academic Officer
Veritas Prep

Looking for GMAT practice questions? Try out the Veritas Prep Question Bank. Learn More.

User avatar
Senior | Next Rank: 100 Posts
Posts: 94
Joined: Tue Jun 30, 2009 4:51 pm
Thanked: 24 times
Followed by:7 members
GMAT Score:750

by havok » Mon Apr 18, 2011 6:06 pm
Great, I got A as well - although I thought my deeming 'non-mathematicians' as false answers felt odd.

Master | Next Rank: 500 Posts
Posts: 184
Joined: Sat Apr 14, 2007 9:23 am
Location: Madison, WI
Thanked: 17 times

by ldoolitt » Mon Apr 18, 2011 6:21 pm
Interestingly enough this is a real happening; it's in reference to the computer proof of the 4 color theorem that some people from UI put forth in the 1970s. Real life knowledge comes in handy!

Oh but yeah I agree that A is the only answer that makes any sense really.

Legendary Member
Posts: 857
Joined: Wed Aug 25, 2010 1:36 am
Thanked: 56 times
Followed by:15 members

by AIM GMAT » Mon Apr 18, 2011 10:41 pm
Thanks Brian , yes you are absolutely right about the source , its from LSAT . I found this question really very intresting , its like solving a puzzle .


@force - You got it right buddy .


You guys nailed it :)
Thanks & Regards,
AIM GMAT

GMAT Instructor
Posts: 1302
Joined: Mon Oct 19, 2009 2:13 pm
Location: Toronto
Thanked: 539 times
Followed by:164 members
GMAT Score:800

by Testluv » Mon Apr 18, 2011 11:07 pm
As Brian says, (A) is a nice rephrase of the first sentence of the passage. In an inference question if you see a choice that restates a part of the passage it is automatically correct.

Don't forget that one can use the Kaplan denial test in inference questions:

If (A) were false, would any part of the passage be contradicted? Well, if (A) were false--if, today, some mathematicians refused to accept computer-based proofs--then the first sentence of the passage would become false. Thus, (A) must be true, and actually if you verified (A) was correct using this test, you wouldn't actually have to look at the other choices.
Kaplan Teacher in Toronto

Legendary Member
Posts: 586
Joined: Tue Jan 19, 2010 4:38 am
Thanked: 31 times
Followed by:5 members
GMAT Score:730

by rohu27 » Tue Apr 19, 2011 11:00 am
Nice one AIM,woke me up completely from my semi sleep state. Bingo, im up for more time now ;)
btw i did end up wth A.